LSAT and Law School Admissions Forum

Get expert LSAT preparation and law school admissions advice from PowerScore Test Preparation.

User avatar
 Dave Killoran
PowerScore Staff
  • PowerScore Staff
  • Posts: 5853
  • Joined: Mar 25, 2011
|
#87996
Complete Question Explanation
(The complete setup for this game can be found here: lsat/viewtopic.php?f=162&t=5831)

The correct answer choice is (C).

If Q and S both visit the same site, then they must visit the first site. This occurs because Q must visit a site before R and T visit a site, and so the third site is unavailable. If Q and S both visit the second site, then from the third rule R and T must visit the third site, and from the last rule S must visit the second or third site. But, this configuration does not leave an executive to visit the first site, and so we can infer that Q and S also cannot visit the second site, and they must then visit the first site.

Additionally, from the second rule, we can then infer that F is not the first site visited because the second rule would be violated. Thus, F must be the second site visited, and from the third rule H must then be the third site visited, leaving M to be the first site visited:

G4-Q19-d1.png

Note that either distribution is still possible, with either three executives visiting the first site (V would be the third executive), or with two executives visiting the third site.

Answer choice (A) can be eliminated because from the discussion F cannot be the first site visited.

Answer choice (B) can be eliminated because H must be visited third.
Answer choice (D) can be eliminated because S must visit the first site.

Answer choice (E) can be eliminated because F is the second site, and from the second rule only one executive visits F.

Thus, answer choice (C) is proven correct by process of elimination.
You do not have the required permissions to view the files attached to this post.
User avatar
 atdale1980
  • Posts: 6
  • Joined: Jan 23, 2021
|
#93406
Am I correct that if you were to complete the remaining executive visits, any of V, R, or T could fill in the remaining spaces? I completed the template when answering the question and put R & T together, but then realized they don't necessarily have to be together to satisfy Rule 3.
 Robert Carroll
PowerScore Staff
  • PowerScore Staff
  • Posts: 1787
  • Joined: Dec 06, 2013
|
#93420
atdale1980,

As long as F is only a single person, any of the remaining people could go anywhere, seemingly, and it is not necessary that R and T be together.

Robert Carroll

Get the most out of your LSAT Prep Plus subscription.

Analyze and track your performance with our Testing and Analytics Package.